Any subcollection $mathcal A_1$ of a locally finite collection $mathcal A$ of subsets of $X$ is locally...












0












$begingroup$


Any subcollection $mathcal A_1$ of a locally finite collection $mathcal A$ of subsets of $X$ is locally finite.



Proof



Let $mathcal A_1$ be any subcollection of $mathcal A$ i.e., $mathcal A_1subset mathcal A.$



Let $xin X$ and $mathcal A$ is locally finite, so there exists a nbhd $U$ of $x$ that intersects only finitely many elements $A_1, A_2, A_3,..., A_n$ of $mathcal A$.



More precisely,$A_icap Uneq phi$,where $1le i le n $.



Since,$mathcal A_1subset mathcal A.$,so if any element of {$A_i:1le i le n$} is in $mathcal A_1,$ so $U$ intersects only finitely many elements of $mathcal A_1$,making it locally finite collection of subsets of $X$.



Please check the proof critically up to here, especially the use of quantifiers.. If there is some scope of improvement in the proof or some elegant method other than this please let me know...



My question is "What if no element of {$A_i:1le i le n$} is in $mathcal A_1$?"










share|cite|improve this question









$endgroup$












  • $begingroup$
    Locally Finite: A collection of subsets of a topological space X is said to be locally finite if each point in the space has a neighborhood that intersects only finitely many of the sets in the collection.
    $endgroup$
    – P.Styles
    Dec 24 '18 at 17:57
















0












$begingroup$


Any subcollection $mathcal A_1$ of a locally finite collection $mathcal A$ of subsets of $X$ is locally finite.



Proof



Let $mathcal A_1$ be any subcollection of $mathcal A$ i.e., $mathcal A_1subset mathcal A.$



Let $xin X$ and $mathcal A$ is locally finite, so there exists a nbhd $U$ of $x$ that intersects only finitely many elements $A_1, A_2, A_3,..., A_n$ of $mathcal A$.



More precisely,$A_icap Uneq phi$,where $1le i le n $.



Since,$mathcal A_1subset mathcal A.$,so if any element of {$A_i:1le i le n$} is in $mathcal A_1,$ so $U$ intersects only finitely many elements of $mathcal A_1$,making it locally finite collection of subsets of $X$.



Please check the proof critically up to here, especially the use of quantifiers.. If there is some scope of improvement in the proof or some elegant method other than this please let me know...



My question is "What if no element of {$A_i:1le i le n$} is in $mathcal A_1$?"










share|cite|improve this question









$endgroup$












  • $begingroup$
    Locally Finite: A collection of subsets of a topological space X is said to be locally finite if each point in the space has a neighborhood that intersects only finitely many of the sets in the collection.
    $endgroup$
    – P.Styles
    Dec 24 '18 at 17:57














0












0








0





$begingroup$


Any subcollection $mathcal A_1$ of a locally finite collection $mathcal A$ of subsets of $X$ is locally finite.



Proof



Let $mathcal A_1$ be any subcollection of $mathcal A$ i.e., $mathcal A_1subset mathcal A.$



Let $xin X$ and $mathcal A$ is locally finite, so there exists a nbhd $U$ of $x$ that intersects only finitely many elements $A_1, A_2, A_3,..., A_n$ of $mathcal A$.



More precisely,$A_icap Uneq phi$,where $1le i le n $.



Since,$mathcal A_1subset mathcal A.$,so if any element of {$A_i:1le i le n$} is in $mathcal A_1,$ so $U$ intersects only finitely many elements of $mathcal A_1$,making it locally finite collection of subsets of $X$.



Please check the proof critically up to here, especially the use of quantifiers.. If there is some scope of improvement in the proof or some elegant method other than this please let me know...



My question is "What if no element of {$A_i:1le i le n$} is in $mathcal A_1$?"










share|cite|improve this question









$endgroup$




Any subcollection $mathcal A_1$ of a locally finite collection $mathcal A$ of subsets of $X$ is locally finite.



Proof



Let $mathcal A_1$ be any subcollection of $mathcal A$ i.e., $mathcal A_1subset mathcal A.$



Let $xin X$ and $mathcal A$ is locally finite, so there exists a nbhd $U$ of $x$ that intersects only finitely many elements $A_1, A_2, A_3,..., A_n$ of $mathcal A$.



More precisely,$A_icap Uneq phi$,where $1le i le n $.



Since,$mathcal A_1subset mathcal A.$,so if any element of {$A_i:1le i le n$} is in $mathcal A_1,$ so $U$ intersects only finitely many elements of $mathcal A_1$,making it locally finite collection of subsets of $X$.



Please check the proof critically up to here, especially the use of quantifiers.. If there is some scope of improvement in the proof or some elegant method other than this please let me know...



My question is "What if no element of {$A_i:1le i le n$} is in $mathcal A_1$?"







real-analysis general-topology proof-verification proof-writing quantifiers






share|cite|improve this question













share|cite|improve this question











share|cite|improve this question




share|cite|improve this question










asked Dec 24 '18 at 17:33









P.StylesP.Styles

1,472927




1,472927












  • $begingroup$
    Locally Finite: A collection of subsets of a topological space X is said to be locally finite if each point in the space has a neighborhood that intersects only finitely many of the sets in the collection.
    $endgroup$
    – P.Styles
    Dec 24 '18 at 17:57


















  • $begingroup$
    Locally Finite: A collection of subsets of a topological space X is said to be locally finite if each point in the space has a neighborhood that intersects only finitely many of the sets in the collection.
    $endgroup$
    – P.Styles
    Dec 24 '18 at 17:57
















$begingroup$
Locally Finite: A collection of subsets of a topological space X is said to be locally finite if each point in the space has a neighborhood that intersects only finitely many of the sets in the collection.
$endgroup$
– P.Styles
Dec 24 '18 at 17:57




$begingroup$
Locally Finite: A collection of subsets of a topological space X is said to be locally finite if each point in the space has a neighborhood that intersects only finitely many of the sets in the collection.
$endgroup$
– P.Styles
Dec 24 '18 at 17:57










1 Answer
1






active

oldest

votes


















2












$begingroup$

Your proof is okay.



You could also formulate it like this:



Collection $mathcal A$ is locally finite if and only if for every $xin X$ a neighborhood $U$ of $x$ exists that satisfies:$${Ainmathcal Amid Acap Uneqvarnothing}text{ is a finite set}tag1$$
From $mathcal A_1subseteqmathcal A$ it follows directly that: $${Ainmathcal A_1mid Acap Uneqvarnothing}subseteq{Ainmathcal Amid Acap Uneqvarnothing}tag2$$so if $mathcal A$ is indeed locally finite then from $(1)$ and $(2)$ it follows directly that also:$${Ainmathcal A_1mid Acap Uneqvarnothing}text{ is a finite set}tag3$$
We conclude that $mathcal A_1$ is also locally finite.



Concerning your question:



If ${Ainmathcal A_1mid Acap Uneqvarnothing}$ is empty (in your words no element of ${A_imid 1leq ileq n}$ is in $mathcal A_1$) then the set mentioned under $(3)$ is empty hence is finite.



So nothing is wrong then with the reasoning.



The less elements a collection $mathcal A$ contains the more likely it is that the collection is locally finite.



In specific the empty collection is locally finite.






share|cite|improve this answer









$endgroup$













  • $begingroup$
    :your last statement made my day...thank you...Merry Christmas!!
    $endgroup$
    – P.Styles
    Dec 24 '18 at 18:49










  • $begingroup$
    Merry Christmas to you as well, of course.
    $endgroup$
    – drhab
    Dec 24 '18 at 18:52











Your Answer





StackExchange.ifUsing("editor", function () {
return StackExchange.using("mathjaxEditing", function () {
StackExchange.MarkdownEditor.creationCallbacks.add(function (editor, postfix) {
StackExchange.mathjaxEditing.prepareWmdForMathJax(editor, postfix, [["$", "$"], ["\\(","\\)"]]);
});
});
}, "mathjax-editing");

StackExchange.ready(function() {
var channelOptions = {
tags: "".split(" "),
id: "69"
};
initTagRenderer("".split(" "), "".split(" "), channelOptions);

StackExchange.using("externalEditor", function() {
// Have to fire editor after snippets, if snippets enabled
if (StackExchange.settings.snippets.snippetsEnabled) {
StackExchange.using("snippets", function() {
createEditor();
});
}
else {
createEditor();
}
});

function createEditor() {
StackExchange.prepareEditor({
heartbeatType: 'answer',
autoActivateHeartbeat: false,
convertImagesToLinks: true,
noModals: true,
showLowRepImageUploadWarning: true,
reputationToPostImages: 10,
bindNavPrevention: true,
postfix: "",
imageUploader: {
brandingHtml: "Powered by u003ca class="icon-imgur-white" href="https://imgur.com/"u003eu003c/au003e",
contentPolicyHtml: "User contributions licensed under u003ca href="https://creativecommons.org/licenses/by-sa/3.0/"u003ecc by-sa 3.0 with attribution requiredu003c/au003e u003ca href="https://stackoverflow.com/legal/content-policy"u003e(content policy)u003c/au003e",
allowUrls: true
},
noCode: true, onDemand: true,
discardSelector: ".discard-answer"
,immediatelyShowMarkdownHelp:true
});


}
});














draft saved

draft discarded


















StackExchange.ready(
function () {
StackExchange.openid.initPostLogin('.new-post-login', 'https%3a%2f%2fmath.stackexchange.com%2fquestions%2f3051482%2fany-subcollection-mathcal-a-1-of-a-locally-finite-collection-mathcal-a-of%23new-answer', 'question_page');
}
);

Post as a guest















Required, but never shown

























1 Answer
1






active

oldest

votes








1 Answer
1






active

oldest

votes









active

oldest

votes






active

oldest

votes









2












$begingroup$

Your proof is okay.



You could also formulate it like this:



Collection $mathcal A$ is locally finite if and only if for every $xin X$ a neighborhood $U$ of $x$ exists that satisfies:$${Ainmathcal Amid Acap Uneqvarnothing}text{ is a finite set}tag1$$
From $mathcal A_1subseteqmathcal A$ it follows directly that: $${Ainmathcal A_1mid Acap Uneqvarnothing}subseteq{Ainmathcal Amid Acap Uneqvarnothing}tag2$$so if $mathcal A$ is indeed locally finite then from $(1)$ and $(2)$ it follows directly that also:$${Ainmathcal A_1mid Acap Uneqvarnothing}text{ is a finite set}tag3$$
We conclude that $mathcal A_1$ is also locally finite.



Concerning your question:



If ${Ainmathcal A_1mid Acap Uneqvarnothing}$ is empty (in your words no element of ${A_imid 1leq ileq n}$ is in $mathcal A_1$) then the set mentioned under $(3)$ is empty hence is finite.



So nothing is wrong then with the reasoning.



The less elements a collection $mathcal A$ contains the more likely it is that the collection is locally finite.



In specific the empty collection is locally finite.






share|cite|improve this answer









$endgroup$













  • $begingroup$
    :your last statement made my day...thank you...Merry Christmas!!
    $endgroup$
    – P.Styles
    Dec 24 '18 at 18:49










  • $begingroup$
    Merry Christmas to you as well, of course.
    $endgroup$
    – drhab
    Dec 24 '18 at 18:52
















2












$begingroup$

Your proof is okay.



You could also formulate it like this:



Collection $mathcal A$ is locally finite if and only if for every $xin X$ a neighborhood $U$ of $x$ exists that satisfies:$${Ainmathcal Amid Acap Uneqvarnothing}text{ is a finite set}tag1$$
From $mathcal A_1subseteqmathcal A$ it follows directly that: $${Ainmathcal A_1mid Acap Uneqvarnothing}subseteq{Ainmathcal Amid Acap Uneqvarnothing}tag2$$so if $mathcal A$ is indeed locally finite then from $(1)$ and $(2)$ it follows directly that also:$${Ainmathcal A_1mid Acap Uneqvarnothing}text{ is a finite set}tag3$$
We conclude that $mathcal A_1$ is also locally finite.



Concerning your question:



If ${Ainmathcal A_1mid Acap Uneqvarnothing}$ is empty (in your words no element of ${A_imid 1leq ileq n}$ is in $mathcal A_1$) then the set mentioned under $(3)$ is empty hence is finite.



So nothing is wrong then with the reasoning.



The less elements a collection $mathcal A$ contains the more likely it is that the collection is locally finite.



In specific the empty collection is locally finite.






share|cite|improve this answer









$endgroup$













  • $begingroup$
    :your last statement made my day...thank you...Merry Christmas!!
    $endgroup$
    – P.Styles
    Dec 24 '18 at 18:49










  • $begingroup$
    Merry Christmas to you as well, of course.
    $endgroup$
    – drhab
    Dec 24 '18 at 18:52














2












2








2





$begingroup$

Your proof is okay.



You could also formulate it like this:



Collection $mathcal A$ is locally finite if and only if for every $xin X$ a neighborhood $U$ of $x$ exists that satisfies:$${Ainmathcal Amid Acap Uneqvarnothing}text{ is a finite set}tag1$$
From $mathcal A_1subseteqmathcal A$ it follows directly that: $${Ainmathcal A_1mid Acap Uneqvarnothing}subseteq{Ainmathcal Amid Acap Uneqvarnothing}tag2$$so if $mathcal A$ is indeed locally finite then from $(1)$ and $(2)$ it follows directly that also:$${Ainmathcal A_1mid Acap Uneqvarnothing}text{ is a finite set}tag3$$
We conclude that $mathcal A_1$ is also locally finite.



Concerning your question:



If ${Ainmathcal A_1mid Acap Uneqvarnothing}$ is empty (in your words no element of ${A_imid 1leq ileq n}$ is in $mathcal A_1$) then the set mentioned under $(3)$ is empty hence is finite.



So nothing is wrong then with the reasoning.



The less elements a collection $mathcal A$ contains the more likely it is that the collection is locally finite.



In specific the empty collection is locally finite.






share|cite|improve this answer









$endgroup$



Your proof is okay.



You could also formulate it like this:



Collection $mathcal A$ is locally finite if and only if for every $xin X$ a neighborhood $U$ of $x$ exists that satisfies:$${Ainmathcal Amid Acap Uneqvarnothing}text{ is a finite set}tag1$$
From $mathcal A_1subseteqmathcal A$ it follows directly that: $${Ainmathcal A_1mid Acap Uneqvarnothing}subseteq{Ainmathcal Amid Acap Uneqvarnothing}tag2$$so if $mathcal A$ is indeed locally finite then from $(1)$ and $(2)$ it follows directly that also:$${Ainmathcal A_1mid Acap Uneqvarnothing}text{ is a finite set}tag3$$
We conclude that $mathcal A_1$ is also locally finite.



Concerning your question:



If ${Ainmathcal A_1mid Acap Uneqvarnothing}$ is empty (in your words no element of ${A_imid 1leq ileq n}$ is in $mathcal A_1$) then the set mentioned under $(3)$ is empty hence is finite.



So nothing is wrong then with the reasoning.



The less elements a collection $mathcal A$ contains the more likely it is that the collection is locally finite.



In specific the empty collection is locally finite.







share|cite|improve this answer












share|cite|improve this answer



share|cite|improve this answer










answered Dec 24 '18 at 18:31









drhabdrhab

103k545136




103k545136












  • $begingroup$
    :your last statement made my day...thank you...Merry Christmas!!
    $endgroup$
    – P.Styles
    Dec 24 '18 at 18:49










  • $begingroup$
    Merry Christmas to you as well, of course.
    $endgroup$
    – drhab
    Dec 24 '18 at 18:52


















  • $begingroup$
    :your last statement made my day...thank you...Merry Christmas!!
    $endgroup$
    – P.Styles
    Dec 24 '18 at 18:49










  • $begingroup$
    Merry Christmas to you as well, of course.
    $endgroup$
    – drhab
    Dec 24 '18 at 18:52
















$begingroup$
:your last statement made my day...thank you...Merry Christmas!!
$endgroup$
– P.Styles
Dec 24 '18 at 18:49




$begingroup$
:your last statement made my day...thank you...Merry Christmas!!
$endgroup$
– P.Styles
Dec 24 '18 at 18:49












$begingroup$
Merry Christmas to you as well, of course.
$endgroup$
– drhab
Dec 24 '18 at 18:52




$begingroup$
Merry Christmas to you as well, of course.
$endgroup$
– drhab
Dec 24 '18 at 18:52


















draft saved

draft discarded




















































Thanks for contributing an answer to Mathematics Stack Exchange!


  • Please be sure to answer the question. Provide details and share your research!

But avoid



  • Asking for help, clarification, or responding to other answers.

  • Making statements based on opinion; back them up with references or personal experience.


Use MathJax to format equations. MathJax reference.


To learn more, see our tips on writing great answers.




draft saved


draft discarded














StackExchange.ready(
function () {
StackExchange.openid.initPostLogin('.new-post-login', 'https%3a%2f%2fmath.stackexchange.com%2fquestions%2f3051482%2fany-subcollection-mathcal-a-1-of-a-locally-finite-collection-mathcal-a-of%23new-answer', 'question_page');
}
);

Post as a guest















Required, but never shown





















































Required, but never shown














Required, but never shown












Required, but never shown







Required, but never shown

































Required, but never shown














Required, but never shown












Required, but never shown







Required, but never shown







Popular posts from this blog

How do I know what Microsoft account the skydrive app is syncing to?

When does type information flow backwards in C++?

Grease: Live!